Mostrar Mensajes

Esta sección te permite ver todos los posts escritos por este usuario. Ten en cuenta que sólo puedes ver los posts escritos en zonas a las que tienes acceso en este momento.

Mensajes - feriva

Páginas: 1 ... 563 564 565 [566] 567
11301
En el presente trabajo que enlazo debajo de este párrafo propongo una demostración de la célebre Conjetura de Goldbach. El método que sigo se basa en las simetrías existentes en un intervalo \( (1, 2n) \) de forma que los elementos, tomando \( n \) como centro de simetría, suman \( 2n \). Se aprovecha la propiedad que tienen los simétricos coprimos con \( 2n \), los cuales se ven obligados a sumar \( 2n \) solamente con compañeros igualmente coprimos con 2n, resultando que esta clase de simétricos coprimos son primos también entre sí. Esto lleva a un análisis que es la clave fundamental del trabajo: los coprimos con 2n no pueden estar respecto de \( 2n \) a una distancia cuyo valor sea divisor de dicho elemento. A partir de aqui surge la observación más curiosa, esto da lugar a la formación de matrices simétricas que aumentan la dimensión de la matriz sucesivamente según se toman pares mayores, sugiriendo ideas interesantes para abordar la hipótesis de Riemann. De ahí, finalmente se concluye que, en general y salvo error mío en algún punto, se cumple la Conjetura.   

http://sitioespacio.a60.us/goldbach_demostracion_matricial.pdf




Enlace interno:

https://foro.rinconmatematico.com/index.php?action=dlattach;topic=

34747.0;attach=6864

11302
Es decir, la traza y el determinante son invariantes del endomorfismo   :laugh:



\(  x(a+c)+y(b+d)=\lambda x+\lambda y

 \)

Y lo que dice Phidias, identificas los coeficientes de las variables de un lado con los coeficientes de las variables en el otro lado, y ya está, no hace falta sacar el polinomio característico. Un método de lo más ortoxo.

No, cuidado, Phidias no está diciendo eso, de esa igualdad no se deduce que \( a+c=\lambda=b+d, \) puesto que \( x \) e \( y \) no son incógnitas, son las coordenadas del vector propio de valor propio \( \lambda. \)

Un saludo.

Hola, Sonata. Gracias por corregirme (aunque soy incorregible porque me equivoco mucho). Es que últimamente he estado trabajando con matrices simétricas, y en mi cabeza daba por hecho que era necesariamente una matriz simétrica. Claro, al ser de orden dos y tener espectro dos...
 Un saludo.

11303
Matemática de Escuelas / Re: Ecuación de primer grado
« en: 20 Junio, 2010, 06:03 am »
Hola

Hola a todos este se me reciste

Calcular \( x \) en:

\( \displaystyle\frac{x-24}{1977}+\displaystyle\frac{x-25}{1976}=\displaystyle\frac{x-1977}{24}+\displaystyle\frac{x-1976}{25} \)

Greetings  ;D

Hola. Espera, verás. Dale a los números grandes el nombre de una letra, que siempre te será menos pesado escribir; y al final cambias las letras por los números y haces las operaciones:

\( 1976=a

1977=b
 \)

\(

{\displaystyle \frac{x-24}{b}+{\displaystyle \frac{x-25}{a}={\displaystyle \frac{x-b}{24}+{\displaystyle \frac{x-a}{25}}}}} \)

Ahora, separa las fracciones que son denominador común en dos fracciones, que se puede hacer perfectamente:

\( {\displaystyle \frac{x}{b}-{\displaystyle \frac{24}{b}+{\displaystyle \frac{x}{a}-{\displaystyle \frac{25}{a}=\frac{x}{24}-{\displaystyle \frac{b}{24}+{\displaystyle \frac{x}{25}-{\displaystyle \frac{a}{25}}}}}}}}  \)

Ahora ya puedes llevarte las equis a un lado, así...

\( {\displaystyle \frac{x}{b}{\displaystyle +{\displaystyle \frac{x}{a}-\frac{x}{24}{\displaystyle -\frac{x}{25}=-{\displaystyle \frac{b}{24}+{\displaystyle -{\displaystyle \frac{a}{25}}}}}}}}+\frac{25}{a}+\frac{24}{b}  \)

Tienen debajo denominador, pero no importa, saca factor común equis de esta manera...

\( x({\displaystyle \frac{1}{b}{\displaystyle +{\displaystyle \frac{1}{a}-\frac{1}{24}{\displaystyle -\frac{1}{25})=-{\displaystyle \frac{b}{24}+{\displaystyle -{\displaystyle \frac{a}{25}}}}}}}}+\frac{25}{a}+\frac{24}{b} \)

Ya sólo tienes que cambiar la \( a \) por 1976 y la \( b \) por 1977; y hacer las cuentas.

 Un saludo.

11304
Hola. Tengo unos problemas con este tema, ya que lo que vi en clase no me ayuda con los ejercicios que se me presentan.

1) Sea \( T:{\mathbb{R}^3}\rightarrow{\mathbb{R}^3}/T(1,0,2)=(0,0,0) ;\textsf{ } T(0,3,0)=(-1,2,1);\textsf{ }T(1,0,0)=(0,0,0) \)
¿Define una transformación lineal?¿Es única?. Justifique su respuesta, y si es afirmativa,  halle la expresión analítica de la transformación lineal.

2) Halle la expresión analítica de una transformación lineal \( T:{\mathbb{R}^3}\longrightarrow{\mathbb{R}^3} \) tal que:
\( Nu(T)=gen{(-2,1,0)} \), \( Im(T): \)vectores posicion incluidos en el plano \( \alpha :x+2y-3z=0 \)

Saludos y gracias

Hola. Tienes que hallar la matriz T, que es el grupo de transformación:

\( \left(\begin{array}{ccc}
a & b & c\\
d & e & f\\
g & h & i\end{array}\right) \left(\begin{array}{ccc}
1 & 0 & 1\\
0 & 3 & 0\\
2 & 0 & 0\end{array}\right) =\left(\begin{array}{ccc}
0 & -1 & 0\\
0 & 2 & 0\\
0 & 1 & 0\end{array}\right)\Longrightarrow\left(\begin{array}{ccc}
a+2c, & d+2f, & g+2i\\
3b & 3e & 3h\\
a & d & g\end{array}\right)=\left(\begin{array}{ccc}
0 & -1 & 0\\
0 & 2 & 0\\
0 & 1 & 0\end{array}\right)

\Longrightarrow a+2c=0; d+2f=-1 ; g+2i=0

3b=0; 3e=2; 3h=0

a=0; d=1 g=0

 \)

Ahora sólo tienes que ir despejando y buscando los valores de a,b,c,d, etc. que son los que forman la base. Miras a ver sin todos linelamente independientes para ver si se trata de una aplicación lineal.

En el segundo problema saca los vectores haciendo: \( x=\lambda; y=\beta \) y ceta lo pones en función de los otros dos parámetros, lambda y beta.
Cuando ya lo tengas das el valor 1 a lambda y el valor cero a beta; con eso sacas el primer vector; que son los factores que multipliquen a lamda (éste sería la cooredenada x) a beta (cooredenada Y) y los coeficientes de la expresión que te haya dado z en función de lambda y beta, que es la cordenada Z.

 Si, por ejemplo, en algún caso te diera \( 2\lambda+3 \) la coordenada del vector sería 2; los sumandos que no están multiplicados por el parámetro se quedan como cero, no se tienen en cuenta.

 Una vez hecho eso ya tienes el primer vector del plano. Para el segundo hace los mismo, pero dando ahora los valores cambiados: a lambda le das valor cero y a beta le das 1.

Las analíticas vienen dadas aquí por estos dos vectores y por un punto del plano, punto el cual sacas multiplicando la matriz que has hallado antes por el último vector que te dan en el ejercicio e igulándolo a cero.

 Un saludo.

11305
Hola, a ver si me pueden ayudar con esto, si (a,b) y (c,d) son filas de la matriz de un endomorfismo cuyos valores propios son 2 y -2, entonces la relación cierta es:
A) a+d=0 ; ad=cb-4 B) a+d=b+2 ; ad=0 C) ad=a+d+2 D) Ninguna es cierta.
En principio se me ha ocurrido hallar el determinante de :
\( \begin{bmatrix}{a-2}&{b}\\{c}&{d+2}\end{bmatrix} \), cuyo resultado es 2a-2d+ad-4-cb pero no veo la relación con claridad. ???

Hola, algebraico. Mira cómo funciona un autovalor. Antes de entenderlo en las matrices, para verlo bien, fíjate en este sistema de ecuaciones:

\( \begin{array}{cc}
ax+by= & \lambda x\\
cx+dy= & \lambda y\end{array} \)

¿Qué puedes deducir al ver esto? Pues que en la primera ecuación, lambda es factor de equis, y en la segunda es factor de \( y \). La solución de cada ecuación no es un término independiente, como en otros sistemas, se expresa en función de la primera variable en la primera ecuación, de la segunda variable en la segunda ecuación, de la tercera en la tercera... Pues bien, ese factor que multiplica a las variables, a la derecha de la igualdad, es un autovalor.

Pero se trata de verlo en matrices; que es lo mismo escrito de otra manera.

Antes de verlo, hay que despejar \( \lambda x \) y \( \lambda y \) igualando así las ecuaciones a cero, y después agrupar los coeficientes de la variable en un paréntesis, o sea, sacas factor común a la variable:.

\( ax-\lambda x+by=x(a-\lambda)+by \)

Y la primera fila de la matriz son los factores de equis e \( y \) de \( x(a-\lambda)+by \)

\( \begin{array}{cc}
(a-\lambda) & b\end{array} \)

Con la segunda ecuación haces lo mismo y sale la segunda fila; y ya tienes la matriz:

\(

\left(\begin{array}{cc}
a-\lambda & b\\
c & d-\lambda\end{array}\right)

 \)

Buenos, pues si vuelves al sistema de ecuaciones que he escrito al principio y sumas las dos ecuaciones...

\(  x(a+c)+y(b+d)=\lambda x+\lambda y

 \)

Y lo que dice Phidias, identificas los coeficientes de las variables de un lado con los coeficientes de las variables en el otro lado, y ya está, no hace falta sacar el polinomio característico. Un método de lo más ortoxo.

Un saludo.

11306
Teoría de Conjuntos / Re: Urgente..
« en: 19 Junio, 2010, 03:20 pm »
Mis saludos...

Necesito un libro sobre teoría de conjuntos. Si me pueden recomendar (enviar link de descarga) alguno. Lo voy agradecer mucho...

Gracias.

Hola. Intenta encontrar "Iniciación a la matemática moderna" de Alfonso Burgos; en España está publicado por editorial Selecciones científicas, pero el matemático que lo escribió es Venezolano, así que puede que esté en más editoriales. Es francamente bueno y agradable de estudiar, muy clarito. Aunque lo mismo está descatalogado, porque es muy antiguo.
 Un saludo.

11307
En el presente trabajo que enlazo debajo de este párrafo propongo una demostración de la célebre Conjetura de Goldbach. El método que sigo se basa en las simetrías existentes en un intervalo \( (1, 2n) \) de forma que los elementos, tomando \( n \) como centro de simetría, suman \( 2n \). Se aprovecha la propiedad que tienen los simétricos coprimos con \( 2n \), los cuales se ven obligados a sumar \( 2n \) solamente con compañeros igualmente coprimos con 2n, resultando que esta clase de simétricos coprimos son primos también entre sí. Esto lleva a un análisis que es la clave fundamental del trabajo: los coprimos con 2n no pueden estar respecto de \( 2n \) a una distancia cuyo valor sea divisor de dicho elemento. A partir de aqui surge la observación más curiosa, esto da lugar a la formación de matrices simétricas que aumentan la dimensión de la matriz sucesivamente según se toman pares mayores, sugiriendo ideas interesantes para abordar la hipótesis de Riemann. De ahí, finalmente se concluye que, en general y salvo error mío en algún punto, se cumple la Conjetura.   

http://sitioespacio.a60.us/goldbach_demostracion_matricial.pdf


11308
Hola, Phidias. Estoy intentando subir un archivo de un trabajo sobre la conjetura, pero no sé dónde se sube. Una proposición de demostración por medio de una serie de matrices espectrales; seguro que te interesará.

Gracias. No obstante, te recomiendo que abrás un nuevo hilo. No es aconsejable mezclar en el mismo temas diferentes. Aún tocando la misma conjetura, no sería razonable mezclarlos.

Saludos.

De acuerdo, muchas gracias a ti. Te paso el enlace del hilo por si quisieras hacerme algún comentario en alguna ocasión:
http://rinconmatematico.com/foros/index.php/topic,34747.msg137716.html#msg137716

 Saludos.

11309
6. ¿Cómo sabe el conductor dónde o cuándo están los números naturales?.

El conductor \( C_t \) cuenta el número natural \( k_0 \) en el instante de tiempo \( \psi_t (k_0)=\hat{k}_0 \).

Saludos.

Hola, Phidias. Estoy intentando subir un archivo de un trabajo sobre la conjetura, pero no sé dónde se sube. Una proposición de demostración por medio de una serie de matrices espectrales; seguro que te interesará. No obstante, lo tengo subido a una de mis páginas:

http://sitioespacio.a60.us/goldbach_demostracion_matricial.pdf

Un saludo.

11310
Sea \( B\equiv{1,2,3,4} \) y sea \( C\equiv{1,2,3,5} \)
Con estos dos conjuntos es imposible establecer una biyección

No es cierto.


Hola, Pepito
Bueo, se puede establecer una biyección tomando subconjuntos de esos conjuntos, pero no entre los conjuntos.
 Un saludo.

11311



.

Quería decir así:

\( A\cap A\equiv A

A\cap A\rightarrow A\Longrightarrow A\rightarrow A \)

Esa condición no la podría cumplir si no fuera suprayectiva, y como es una inyección, es biyectiva.

11312
Sea \( A \) un conjunto finito y \( f:A\rightarrow{}A \), \( f \) es función  , probar que si f es inyectiva entonces es sobreyectiva
Yo he intentado probarlo por inducción sobre el cardinal de \( A \) , \( n(A) \)
si \( n(A)=1 \) entonces \( f(x)=x \) de donde claramente \( f \) es sobreyectiva, mi problema es  como plantear la hipótesis inductiva y de esta llegar a la conclusión pedida
Hola Kurt.  No debe de ser complicado si piensas que todos los elementos de A están en A; eso es lo que significa suprayección. Si \( f \) es inyectiva signfica que a cada elemento de \( A \) origen le corresponde un elemento distinto de \( A \) imagen y que ninguno se queda sin correspondencia (si no, no es una aplicación, es sólo una realción). Una biyección es una aplicación donde se dan ambas cosas: la suprayección y la biyección.
Es importante darse cuenta de esto:
Sea \( B\equiv{1,2,3,4} \) y sea \( C\equiv{1,2,3,5} \)

Con estos dos conjuntos es imposible establecer una biyección; porque 5 está en C pero no en B; y con eso basta, ya no puede ser suptrayectiva y, por tanto, nunca, puede ser biyectiva. Si, en cambio, ocurre que hay tantos elementos en C como en B y, además, todos los que están en C están en B, entonces resulta imposible establecer una inyección no biyectiva.
 La verdad es que es tan simple que cuesta pensar en una demostración; no se puede demostrar que es suprayectiva ya que la cosa se encuentra en la misma definición de suprayección \( A\subset{A} \).
Quizá así:

\( A\cap A\equiv A

A\cap A\rightarrow A\Longrightarrow A\cap A \)

Un saludo


11313



.

11314
Álgebra Lineal (Espacios Vectoriales) / Re: Diagonalización
« en: 18 Junio, 2010, 02:29 am »
Buenas,

de nuevo con dudas, ahora con este ejercicio de diagonalización:

Sea la matriz \( A=\begin{bmatrix}{-1}&{0}&{0}\\{a}&{1}&{0}\\{1}&{0}&{1}\end{bmatrix} \)

a) Determinar los valores propios de A
b) Analizar los valores del parámetro a para que A sea diagonalizable.
c) Para a=2 encuentra una matriz diagonal D y una matriz P tales que \( A=PDP^{-1} \)


Lo que tengo:

a) Para encontrar los valores propios resuelvo la ecuación   \( \left | A-\lambda I \right |=0  \) y obtengo dos valores propios \( \lambda = 1 \) (multiplicidad doble) y \( \lambda = -1 \) (multiplicidad simple)

b) Para ver cuando es diagonalizable hago \( (A-\lambda I) X=0 \)

Para \( \lambda=-1 \)
\( \begin{bmatrix}{0}&{0}&{0}\\{a}&{2}&{0}\\{1}&{0}&{2}\end{bmatrix} \left[{\begin{array}{ccc}{x}\\{y}\\{z}\end{array}\right] = 0 \), de donde obtengo que la imagen de f es\(  \left[{\begin{array}{ccc}{-2\alpha}\\{-a\alpha}\\{\alpha}\end{array}\right] \) y una base \(  \left[{\begin{array}{ccc}{-2}\\{-a}\\{1}\end{array}\right] \)


Para \( \lambda=1 \)
\( \begin{bmatrix}{-2}&{0}&{0}\\{a}&{0}&{0}\\{1}&{0}&{0}\end{bmatrix} \left[{\begin{array}{ccc}{x}\\{y}\\{z}\end{array}\right] = 0 \), de donde obtengo que la imagen de f es\(  \left[{\begin{array}{ccc}{0}\\{0}\\{0}\end{array}\right] \)

En principio, la forma que tengo de ver si es diagonalizable es sumando las dimensiones de las imágenes que obtengo (en este caso 1 ambas), si son iguales al rango de A, entonces la matriz es diagonalizable. Algo debo de estar haciendo mal porque no entiendo en que cambia "a" las cosas... tenga el valor que tenga la dimensión de A es 3, no?

Cualquier ayuda es bienvenida.

Un saludo y gracias

Hola, Harpo. Te has equivocado al restar los autovalores en la diagonal; fíjate en los signos, buen hombre.  ;)

Otra cosa, después de hallar cada  autovector -con la equivocación que te he señalado- dices: "una base es..." Un vector suelto no es una base, es el conjunto de ellos, siempre y cuando sean linealmente independientes, claro, lo que quiere decir que sus coordenadas no son proporcionales; por ejemplo:

(2,3,5) y (4,6,10) son línealmente independientes porque las coordenadas del segundo son las del primero multiplicadas por 2, ¿lo ves? Esos dos vectores, en el espacio, dibujan unas paralelas; los no linealmente independientes en cambio nunca lo hacen. Así, cuando tú tienes los ejes de coordenadas en tres dimensiones, ves que ninguno de ellos es paralelo; en esto puedes visualizar lo que significa una base en un espacio de tres dimensiones.
Si tienes tres vectores de tres coordenadas y con dos de ellos pasa eso, que te encuentras con dos vectores linealmente dependientes (de coordenadas proporcionales según una misma razón) es que no forman una base.

 Entonces ¿qué tienes que hacer? Primero, tener cuidado con los signos, y una vez sacados los autovectores, mirar si alguno de ellos tiene las coordenadas proporcionales; en ese caso no será una base.
En este problema tienes un valor que es "a", el cual tiene que tener un valor o unos valores que evite esto que te digo para que los tres vectores sean independientes y formen una base.
Para solucionar estas cosas, recuerda también que las matrices simétricas son siempre diagonalizables.
 Un saludo.



11315
Foro general / Re: Autoria e investigación en la red.
« en: 16 Junio, 2010, 11:29 pm »
El -manco 
Tal vez lo publique aquí pero estoy haciendo una web, tengo  una hecha pero de otros temas
Talvez no sea un matemático  y solo puedo decir que tengo ideas ,y que el mundo matemático no es central en mis investigaciones,
El hecho es  que apartir de una una  investigación que hice tratando de hallar la relación entre un cuadrado y el circulo lo cual me daba infinito, no  se podía
Probé con los logaritmos ,para ver si lograba un acercamiento pero aquí me encontré con cosas raras   y mis dudas son
Porque se usan calculadoras para lograr la mantisa
Porque se usa el numero de Euler y es tan valioso
Porque no hay otro método aparte del calculo para obtener la mantisa
Yo encontré algo pero me párese raro todo esto porque , es muy simple y temo estar cayendo en la ingenuidad.

Entonces te pregunto 

Si

   3,52525
10         

¿No conoces ningún otro método para obtener la mantisa?

Dado que hay una pregunta que yo hice en este foro  que se relaciona con esto y que do dando vueltas.

http://rinconmatematico.com/foros/index.php/topic,23747.msg95311.html#msg95311

y quizás cuantas cosas mas.

Se que cualquier cosa tiene que ser probada en rigor y dado mi escaso nivel matemático  me es difícil hacerlo .entonces también podría ser prematuro hablar de ello


Hola, Zonurb1. Yo registro cualquier clase de cosa en safecreative.org. Es muy fácil, te das de alta y subes el tipo de archivo que quieras.

 Un saludo.

11316
Gracias por tu respuesta, la verdad es que sí, hace unos dias lo pensé y razoné que con cada primo que nos encontramos de más las combinaciones se doblan, y creo que no llegará a superar el límite del intervalo (\( b \)). Este tema de los primos es muy curioso, y pensar que el 64 solo es un múltiple de un primo, como cualquier otro, para mi, solo existen los primos como números, el resto, son múltiples.

Hola, Iluminatus. No le des más vueltas, la conjetura se cumple con casi toda seguridad, lo que ocurre es que aún no se ha admitido ningún intento de demostración de los muchos que ha habido; como te gusta pensar las cosas por ti mismo, sólo te digo que busques en Google "Vinográdov", "postulado de Bertrand", busca también "hipótesis de Riemann", lo que implica ésta y lo que piensan los mejores matemáticos que la están estudiando sobre si se cumple o no.
 Un saludo.


11317


 
Los axiomas no necesitan demostración...

Estás interpretando erróneamente el sentido de esa frase.
Criticás esa frase diciendo que "no queremos demostrar".
O sea, que según vos, "quienes no dan la demostración de un axioma es porque lo dan por obvio".

O sea, la palabra axioma sería, según estás entendiendo, sinónimo de "teorema obvio".

Pero la cosa no va por ese lado.
Los axiomas no se dan solos, sino en bloque.

Supongamos que queremos desarrollar una cierta teoría matemática.
Se declara una lista de símbolos vacíos de significado, tal como \( 1, N, s, < \).
A tales símbolos sin significado se les llama elementos primitivos de la teoría.
A continuación se lista una serie de propiedades o relaciones que han de cumplir dichos elementos primitivos.
Esta lista de propiedades serían los axiomas.
La función de la lista de axiomas es proporcionar el punto de partida de una teoría matemática.
Podrías pensarlo como una lista de hipótesis de los futuros teoremas.

Un teorema tiene la forma siguiente: si se cumple la hipótesis H entonces es válida la conclusión o tesis T.

Los axiomas son como hipótesis que aparecen en todo teorema subsecuente, en la teoría que estamos desarrollando.
La única salvedad es que, por brevedad, dichas hipótesis/axiomas ahora no se escriben, porque sería una repetición molesta e innecesaria.
Pero así es como conviene entender los axiomas.

Ahora bien, si a los signos \( 1, N, s, < \), les damos los axiomas de Peano, da lugar a un sistema que conocemos como "los números naturales".
Que el 1 sea el "primer elemento" quiere decir que en N hay un buen orden, por lo tanto tiene primer elemento, y que ese primer elemento se lo "denota" con 1.
El "concreto" axioma sería que N tiene un buen orden, y en tal caso tiene que tener un elemento que es el primero. Si a ese elemento se lo llama "1", poco importa, es sólo un signo que se usa para denotar un objeto del que se tiene certeza que existe.

Ahora bien.
Hay otros sistemas axiomáticos, que dan lugar a los enteros, los racionales, los reales, etc.
Cada uno de esos sistemas incluye como subsistema a los naturales.
En estos nuevos sistemas el 1 no es, claro está primer elemento.
Pero cuando se considera el subconjunto de naturales que hay en cada uno de ellos, lo que queda es un sistema que cumple todas las propiedades que uno espera de los naturales, o sea, satisface los axiomas, y por lo tanto tiene un primer elemento.
Se puede probar que el 1 es dicho primer elemento en ese caso.
Pero siempre restringiéndose a N.

Pareciera que al introducir N con axiomas, uno estaría "creando a los naturales de la nada".
Pero eso no es así.
Uno puede construir un "ejemplo" de conjuntos y elementos que satisfacen los axiomas.

Dicho más claramente, si yo hablo de los astros grises y demuestro una serie de propiedades y doy una lista de axiomas... la teoría puede estar correcta, pero podría ser que no haya astros grises.
Sería una teoría "sin aplicación a casos reales".
Pero cuando digo: la Luna es un astro gris, le puedo aplicar a la Luna toda mi teoría, y entonces los axiomas tienen al menos un caso de aplicación, y en tal caso son consistentes.

Para los naturales pasa lo mismo. Se puede construir un sistema que cumpla sus propiedades.
Incluso se pueden construir infinidad de sistemas que cumplan esas propiedades.
Y lo más curioso de todo es que todos esos sistemas son equivalentes entre sí, en sentido algebraico y ordinal.
Por lo tanto, uno puede pensar que, en el fondo, hay un solo sistema de números naturales.

Distinto es el caso de los axiomas de Grupo: hay muchos grupos que no son equivalentes entre sí.
Pero sistemas "N" de naturales... son todos lo mismo, salvo isomorfismos.

Para más detalles sobre la construcción de los números, te invito a revisar el siguiente thread:

Construcción de los Sistemas Numéricos



Hola, Argentinator, muchas gracias por el comentario.
 Tengo bastante olvidado lo muy poco de análisis que sabía y te agradezco el recordatorio (tengo olvidado casi todo de todo).
Fíjate en que no he negado que tal axioma pertenezca a un sistema lógico ideado para demostrar propiedades dentro de un conjunto, simplemente he dejado de mencionarlo (esto es sólo por justificar mi error con una broma ).

Porque tienes mucha razón, porque no me estaba refiriendo a un tipo de axioma cualquiera, sino al primero dentro de todos los axiomas de Peano, al más elemental, y basándome en él he generalizado; lo he utilizado con el significado según la acepción lingüística, no matemática, que en el caso del primer axioma del sistema sí se puede hacer. No era ni mucho menos una crítica, lo decía con un poco de ironía y no sólo referido a las matemáticas, quería hacer ver la frustración que, a menudo, nos deja el no poder llegar a un por qué más básico.

Un cordial saludo y muchas gracias otra vez.

11318
Hola! Os explico:

Hace meses estaba investigando sobre el patrón de los primos y la Conjetura de Goldbach, "tocando" por así decirlo cada tema. Navegando por Internet encontré una fórmula \( 2^x^-^1 -1 \) donde \( x \) es número primo. Si el resultado era divisible por \( x \) entonces \( x  \) es primo.

La cuestión es: ¿De donde salió esa formula? ¿Hay alguna demostración?

Gracias.


Hola, es de Fermat; es lo que se llama "el pequeño teorema de Fermat", si no me equivoco.
Un saludo.

11319


La pregunta tiene mucho más sentido del que aparenta, y lo tiene porque el axioma que dice "el uno es el primero" sólo es cierto si está referido a los números naturales, a los números del conjunto \(  \mathbb{N} \). Para verlo, tomemos otro conjunto, el conjunto de los números enteros,  \(  \mathbb{Z} \), que es igual que el otro pero incluyendo, además de los anteriores, los enteros negativos. Ahora la afirmación no es cierta, el \( 1 \) ya no es el primero. Quién es el primero ahora.
 Si entendemos la pregunta como cualquiera la entendería (cualquiera que no fuera un político)  el primero es el que está más a la izquierda según escribimos la sucesión. Y resulta que hacia ese lado encontramos, como vistos en un espejo, todos los elementos que hay a la derecha, pero con una rayita delante: \(  -1,-2-3,...-n \). El primero, en negativo, es tan grande  como queramos, o sea, siempre se puede encontrar uno "más menor". Ocurre lo mismo análogamente que con el último por la derecha; que siempre puede ser "más mayor".

El axioma no funciona en  \(  \mathbb{Z} \), es falso; eso sí, es tan falso que tal falsedad no necesita demostración (algo es algo).
Se dice que un axioma es una afirmación que no necesita ser demostrada; a mí me gusta decir que sí lo necesita, pero que no somos capaces de hacerlo (mas quisiéramos poder hacerlo; y también se podría exclamar, con un "más" no adversativo, ¡más quisiéramos!)

Es complicado o prácticamente imposible encontrar un axioma que funcione respecto de todas las propiedades matemáticas involucradas en los distintos conjuntos numéricos; visto el ejemplo anterior cualquiera puede imaginar otros casos. Aún nos daremos más cuenta de esto si no sólo pensamos en números pelados y mondados: escalares, valores asociados a una sola coordenada, para entendernos. La cosa se complica si usamos un elemento como éste:

 \( (1,-3,5) \)

Si nos referimos al primer cuadrante del sistema de coordenadas, sí podemos decir que ese 1 es el primero a partir del origen... pero es sólo un componente del elemento, una parte de él, y, por ende, hay más cuadrantes.   

¿Cómo podríamos encontrar el primer número de todos, el primero de verdad? No se puede, y ahora veremos por qué.

 Sin meternos en números complejos, busquemos el primer número real por la izquierda; pasa algo parecido a lo que ocurría antes, siempre hay uno menor que el anterior, lo mismo que siempre hay uno más grande por la derecha.

No obstante, ¿por qué no decir que el 1 es ese primer número real, ése tan pequeño?
El primer problema que encontramos, el más obvio, es que, al no estar definido, no nos va a servir bien de elemento neutro; el uno es neutro porque cuando multiplicamos o dividimos por él, el número sobre el que actúa se queda igual que estaba:
\(  1a=a \)

Claro, las complicaciones que van a surgir si asignamos por decreto esta propiedad al cero, o casi cero, van a ser demasiadas (se nos quedaría todo en nada, imaginaos las cuentas de los bancos, encima de la crisis que hay).
Bien, aun así, podríamos intentar hacerlo por convención. Ya está el "casi cero" es el primero. Qué bien, ya no vamos a necesitar números racionales, ni reales...
 Así que al casi cero lo designamos con este símbolo 1, al siguiente con 2...

Y ¿qué va a pasar? Va a pasar que va a haber números que no van a ser múltiplos de ninguno de los números anteriores, o sea, no van a ser múltiplos de ningún número menor que ellos; salvo del primero, por la mencionada propiedad de neutralidad que le hemos otorgado para hacer este experimento. Esos números serán, serían, los primos; y habría primos que tenderían a cero, según este invento.

 Siendo así, cuando fuéramos a dividir, a partir en partes iguales, los mencionados números por otra cantidad... nos aparecerían siempre, en todos los casos, los números \(  \mathbb{Q}  \) , los racionales.

Ellos son los principales responsables de que aparezcan los números no enteros, ellos, los de siempre, los números primos. Los primos se presentan tarde o temprano (más bien tarde y temprano) aparecen  en algún lugar de la sucesión por primera vez (por eso se llaman primos) y cada uno de ellos, particularmente, no vuelve a aparecer nunca más.

En fin, y por esto el uno, nuestro uno, no puede ser realmente (\(  \mathbb{R}  \)ealmente) el primero.

11320
Foro general / Re: Ayer fue ayer, luego no soy un iconoclasta.
« en: 15 Junio, 2010, 01:28 am »
Era un profesor bastante joven,se llamaba José Luis

Lo de ser joven se corrige con el tiempo, lo de llamarse Jose Luis, con el Registro Civil  :laugh:.

Saludos.

  :laugh: Comprendo.

Un saludo y buenas noches.

Páginas: 1 ... 563 564 565 [566] 567